Đến nội dung

Hình ảnh

VMF's Marathon Hình học Olympic

* * * * * 1 Bình chọn hình học

Lời giải halloffame, 02-01-2018 - 16:29

Lời giải bài toán 196. Ta chứng minh bài toán cho đường tròn $(K)$ tiếp xúc trong $(O),$ trường hợp tiếp xúc ngoài chứng minh tương tự. Ta thấy có thể bỏ đi điểm $B$ không cần thiết.

Bài toán 196'. $\Delta ADC$ vuông tại $D$ nội tiếp $(O),$ một đường tròn $(E)$ tiếp xúc trong $(O)$ ở $T.M,N \in (E)$ sao cho $MN \parallel AD$ và $MN=AD.P,R$ là trung điểm $MD,MC.$ Khi đó $P \in (ORT).$

Chứng minh. 

$M'$ đối xứng $M$ qua $T.$ Dựng điểm $I$ sao cho $OEMI$ là hình bình hành.

$OI$ cắt $(O),CD$ ở $K,L.J$ là hình chiếu $I$ lên $CD.$

Từ $OEMI$ là hình bình hành và $EM=ET$ ta suy ra được $IK=IM=JN,LK=LM$

Gọi $Q$ đối xứng $M$ qua $O$ thì $Q \in LM'.$ Ta có $LM'.LQ=LK.NJ=LK.KI=KO^2-OL^2=LC.LD \Rightarrow Q \in (M'CD).$

Qua phép vị tự tâm $M$ tỉ số $\frac{1}{2}$ ta có ngay đpcm.

[attachment=33194:Screen Shot 2018-01-02 at 1.29.42 AM.png]

Đi đến bài viết »


  • Please log in to reply
Chủ đề này có 434 trả lời

#81
quanghung86

quanghung86

    Thiếu úy

  • Điều hành viên
  • 632 Bài viết

Được sự đồng ý của Dương, mình đề nghị hộ một bài toán

 

$\boxed{\text{Bài toán 39}}$ (Telv Cohl+Trần Quang Hùng). Cho tam giác $ABC$ và $P$ là một điểm trên đường tròn ngoại tiếp. $Q$ là hình chiếu của $P$ lên cát tuyến trực giao của $P$ ứng với tam giác $ABC$. Chứng minh rằng trực tâm các tam giác $APQ,BPQ,CQP$ cùng nằm trên đường thẳng $\ell$ và đường thẳng qua $P$ vuông góc với $\ell$ luôn đi qua điểm cố định khi $P$ thay đổi.

 

Khái niệm cát tuyến trực giao


Bài viết đã được chỉnh sửa nội dung bởi quanghung86: 16-06-2016 - 19:24
Nhầm số bài


#82
baopbc

baopbc

    Himura Kenshin

  • Thành viên nổi bật 2016
  • 410 Bài viết

$\boxed{\text{Lời giải bài toán 39}}$

Post 214.png

Trước tiên, ta sẽ chứng minh $\ell $ song song với đường thẳng $Gauss$ của tứ giác toàn phần $ABCDEF$.

Giả sử $D,E,F$ theo thứ tự là giao của đường thẳng qua $P$ vuông góc với $PA,PB,PC$ với $BC,CA,AB$.

$A'\equiv PD\cap (O),C'\equiv PF\cap (O)$. Do $\angle APA'=\angle CPC'=90^\circ$ nên $\overline{A,O,A'}$ và $\overline{C,O,C'}$.

Áp dụng định lí $Pascal$ ta suy ra $O$ thuộc $DF.M,N$ theo thứ tự là trung điểm $AD,CF.H_a,H_c$ lần lượt là trực tâm tam giác $APQ$ và $CPQ$

Do $AH_a\parallel CH_c\parallel FD,M,N$ là trung điểm $AD,CF$ nên theo định lí $Thales$ ta chỉ cần chỉ ra $AH_a+CH_c=FD$.

Gọi $O_a,O_c$ theo thứ tự là tâm ngoại tiếp các tam giác $APQ$ và $CPQ.OO_a,OO_c$ cắt đường thẳng qua $P$ song song với $DF$ tại $S,T$.

Theo tính chất hình bình hành thì $ST=FD$ nên theo định lí về đường trung bình thì $FD=2O_aO_c$ hay $AH_a+CH_c=FD$.

$\Rightarrow H_aH_c\parallel MN$. Theo định lí về đường thẳng $Gauss$ thì $H_a,H_b,H_c$ thẳng hàng hay $\ell$ song song với đường thẳng $Gauss$ của tứ giác toàn phần $ABCDEF$.

Gọi $H$ là trực tâm của tam giác $ABC$, dễ thấy $H$ thuộc trục đẳng phương của $(AD),(BE),(CF)$. Mặt khác $P\equiv (AD)\cap (BE)\cap (CF)$ nên $PH$ là trục đẳng phương của $(AD),(BE),(CF)\Rightarrow PH\perp \overline{M,N,P}$ hay $PH\perp \ell\Rightarrow $ đường thẳng qua $P$ vuông góc với $\ell$ luôn đi qua một điểm cố định. $\blacksquare$

Bài toán đề nghị. $\boxed{\text{Bài toán 40}}$ (Thầy Trần Quang Hùng) Cho tứ giác $ABCD$ nội tiếp đường tròn $(O).E,F$ lần lượt là trung điểm $AB,CD.AC$ cắt $BD$ tại $P.G$ là ảnh của $P$ qua phép nghịch đảo tâm $O.Q,R$ là hình chiếu của $G$ lên $AB,CD.K,L$ là các điểm trên đường thẳng qua $P$ song song với $AD$ sao cho $EL\perp PQ,FK\perp PR$. Chứng minh rằng $ADKL$ là hình bình hành.

Post 215.png

$$\begin{array}{| l | l |} \hline Ngockhanh99k48 & 8\\ \hline IHateMath & 1\\ \hline fatcat12345 & 5\\ \hline dogsteven & 5\\ \hline baopbc & 10\\ \hline QuangDuong12011998 & 5\\ \hline xuantrandong & 2\\ \hline mrjackass & 1\\ \hline vietnaminmyheart & 3\\ \hline BuiBaAnh & 1\\ \hline halloffame & 2\\ \hline\end{array}$$


Bài viết đã được chỉnh sửa nội dung bởi baopbc: 20-06-2016 - 18:22


#83
quanghung86

quanghung86

    Thiếu úy

  • Điều hành viên
  • 632 Bài viết

Cám ơn Bảo, lời giải rất hay khác với lời giải thầy biết, sau đây là link gốc của bài toán

 

http://www.artofprob...nity/c6h1144762



#84
Nguyen Dinh Hoang

Nguyen Dinh Hoang

    Hạ sĩ

  • Thành viên
  • 99 Bài viết

$\boxed{\text{Lời giải bài toán 40}}$

$\boxed{\text{Bổ đề}}$ (Đường thẳng $Steiner$ của tứ giác toàn phần)

Cho tứ giác toàn phần $ABCD,PQ$ Trực tâm của các tam giác sau $PAB$, $PCD$, $QAD$, $QBC$ thẳng hàng và đường thẳng này đi qua giao điểm 2 đường chéo

Trở lại bài toán.

Post 219.png

Gọi $S$, $T$ lần lượt là điểm đối xứng của $G$ qua $Q$ và $R$ theo bổ đề thì $ST$ đi qua $P$. Từ $P$ kẻ $PH$ vuông góc $AD$ ($H$ thuộc $GS$ )

Gọi $Z$ là hình chiếu của $P$ lên $AB$, $Lx\parallel AB$

Ta sẽ chứng minh $P(ZG,QH)$ $=$ $L(xA,EP)$.

Đúng vì chiếu $P(ZG,QH)$ $=$ $P(G,QH)(1)$ và $L(xA,EP)=L(A,EW)$(2) trong đó W là giao của $PK$ với $AB$

Mặt khác ta có $S$ và $G$ đối xứng nhau qua $Q$.Gọi $K$ là đối xứng của $G$ qua $AD$ thì $G$, $P$, $S$ thẳng hàng và sử dụng định lý hàm số $Cos$ ta dễ dàng chứng minh được $BS\parallel KD$.

Nên $\frac{PS}{PK} = \frac{BP}{PD} \Rightarrow \frac{HS}{GH} = \frac{PB}{PD}(3)$

Nên từ (1) (2) (3) ta có $AL$ vuông góc $PG$ theo tính chất hàng điểm vuông góc.

Chứng minh tương tự ta suy ra $AL\parallel DK$ hay ta có $đ.p.c.m$

$$\begin{array}{| l | l |} \hline Ngockhanh99k48 & 8\\ \hline IHateMath & 1\\ \hline fatcat12345 & 5\\ \hline dogsteven & 5\\ \hline baopbc & 10\\ \hline QuangDuong12011998 & 5\\ \hline xuantrandong & 2\\ \hline mrjackass & 1\\ \hline vietnaminmyheart & 3\\ \hline BuiBaAnh & 1\\ \hline halloffame & 2\\ \hline Nguyen Dinh Hoang & 1\\ \hline\end{array}$$


Bài viết đã được chỉnh sửa nội dung bởi baopbc: 20-06-2016 - 19:05


#85
quanghung86

quanghung86

    Thiếu úy

  • Điều hành viên
  • 632 Bài viết

Lời giải bài toán 40.

 

Figure3879.png

 

Ta sẽ chứng minh $AL$ và $DK$ cùng vuông góc với $OG$ thì $ADKL$ là hình bình hành, thật vậy. Đường thẳng qua $P$ vuông góc $OP$ cắt $BC,AD$ tại $S,T$. Theo bài toán con bướm thì $P$ là trung điểm $ST$. Gọi $BC$ cắt $AD$ tại $H$. Theo tính chất điểm Miquel dễ thấy tứ giác $ABGH$ nội tiếp nên $\angle BGA=\angle BHA$ và $\angle GBA=180^\circ-\angle GAH=\angle HTS$. Từ đó hai tam giác $GBA$ và $HTS$ đồng dạng g.g. Chú ý $GQ$ là đường cao của tam giác $GBA$ nên nếu gọi $HW$ là đường cao tam giác $HTS$ nên $\frac{PG}{ST}=\frac{HW}{ST}=\frac{GQ}{AB}$. Từ đó suy ra $\frac{GP}{GQ}=\frac{ST}{AB}=\frac{AL}{AE}$. Gọi $M$ là hình chiếu của $G$ lên $PQ$ và $N$ là hình chiếu của $A$ lên $EL$. Do $EA\perp QG,EL\perp PQ$ nên hai tam giác vuông $ANE$ và $GMQ$ đồng dạng. Suy ra $\frac{AN}{GM}=\frac{AE}{GQ}=\frac{AL}{GP}$, từ đó hai tam giác vuông $ALN$ và $GPM$ đồng dạng, suy ra $AL\perp GP$.

 

$\boxed{\text{Bài toán 41.}}$ Cho tam giác $ABC$ có $P,Q$ là hai điểm đẳng giác. $K,L$ là tâm đường tròn $(PBC),(QBC)$. $PK,QL$ lần lượt cắt trung trực $AP,AQ$ tại $M,N$. $E,F$ là hình chiếu của $M$ lên $CA,AB$. $G,H$ là hình chiếu của $N$ lên $CA,AB$. Chứng minh rằng $E,F,G,H$ cùng thuộc một đường tròn và đường tròn này luôn tiếp xúc một đường thẳng cố định khi $P,Q$ thay đổi.

 

Figure3880.png


Bài viết đã được chỉnh sửa nội dung bởi baopbc: 20-06-2016 - 18:43
Sửa lại số vì bài toán 41 đã được ẩn


#86
viet nam in my heart

viet nam in my heart

    Thượng sĩ

  • Điều hành viên OLYMPIC
  • 242 Bài viết

Mình có một đề xuất nhỏ thế này:

Sau mỗi bài toán có thể đừng đề xuất bài toán mới vội. Hãy để mọi người cùng kiểm tra và sửa lại lỗi trong bài đó (Mặc dù giải hình rất ít khi nhầm lẫn nhưng việc đưa lời giải lên mạng thì sai sót có rất nhiều). Người giải nếu không có thời gian thì có thể nhắn tin bài toán mới đó cho Bảo. Sau khi chắc chắn lời giải đó đúng thì hẵng đưa bài toán mới lên. Nhờ Bảo sau mỗi lời giải bài toán có thể vẽ chỉnh lại hình cho dễ quan sát được không

Như trong lời giải vừa rồi của bạn Hoàng đã gõ sai khá nhiều lỗi. Hơn nữa việc đưa ra bổ đề nó đúng trong trường hợp tứ giác nội tiếp chứ không phải tất cả và việc áp dụng cũng cần chỉnh lại là áp dụng ở đâu. 


Bài viết đã được chỉnh sửa nội dung bởi viet nam in my heart: 20-06-2016 - 18:47

"Nếu bạn hỏi một người giỏi trượt băng làm sao để thành công, anh ta sẽ nói với bạn: ngã, đứng dậy là thành công." Isaac Newton

VMF's Marathon Hình học Olympic


#87
quanghung86

quanghung86

    Thiếu úy

  • Điều hành viên
  • 632 Bài viết

Ủng hộ ý kiến của viet nam in my heart, chúng ta nên dành thêm chút thời gian để đọc và phản biện lời giải đưa lên.



#88
TQHKTH

TQHKTH

    Binh nhì

  • Thành viên mới
  • 11 Bài viết

Lời giải của mình cho bài toán 41.

Bổ đề 1. Cho $\triangle ABC$ và bộ 6 điểm $M, N\in BC,\ P, Q\in CA,\ R, S\in AB$ sao cho $MNPQ, PQRS, RSMN$ là 3 tứ giác nội tiếp. Khi đó 6 điểm trên đồng viên (quen thuộc và dễ CM).

Bổ đề 2.  Cho $P, Q$ đẳng giác trong $\triangle ABC$. Gọi $S$ là tiếp điểm trên $BC$ của inellipse nhận $P, Q$ là tiêu điểm. Khi đó:

$\angle PSB = \angle QSC = \angle PCB+\angle QBC+\angle PAB$

Vào bài. Kí hiệu $\alpha = \angle PCB+\angle QBC+\angle PAB$

Đầu tiên, dễ dàng cộng góc để có $\angle PAM = \angle QAN = 90^\circ – \alpha$. Do đó $AM, AN$ đẳng giác trong $\angle BAC$, dẫn đến $E, F, G, H$ đồng viên. Tuy nhiên, điều cốt yếu trong bài toán này nằm ở việc CM $EFGH$ tiếp xúc đường trung bình đỉnh $A$ của $\triangle ABC$, và bây giờ ta sẽ CM nó.

Vị tự $\mathcal{H}_A^2$ và phát biểu lại bài toán:

“Cho $\triangle ABC$ có $P, Q$ đẳng giác. Trên đường thẳng qua $P$ vuông góc $PA$ lấy $I$ sao cho $\angle PIA = \alpha$. Tương tự có điểm $J$. $X, Y$ là hình chiếu $I$ lên $CA, AB$ và $Z, T$ là hình chiếu $J$ lên $CA, AB$. CMR: $(XYZT)$ tiếp xúc $BC$.”

Giải bài. Gọi $S$ là tiếp điểm trên $BC$ của inellipse nhận $P, Q$ là tiêu điểm. Khi đó theo bổ đề:

$\angle PSB = \angle QSC = \alpha$

$\Rightarrow \angle PSB = \angle PIA = \angle PYA$, nghĩa là $SPYB$ nội tiếp.

Hoàn toàn tương tự ta có: $SQZC, SPXC, SQTB$ cũng nội tiếp.

Bây giờ: $\angle PSB = \alpha = \angle QJA = \angle QZA$ đồng thời $\angle PCS = \angle QCZ$, thành thử $\angle CPS = \angle CQZ$

$\Rightarrow \angle CXS = \angle CPS = \angle CQZ = \angle CSZ \Rightarrow (SXZ)$ tiếp xúc $BC$.

Tương tự $(SYT)$ cũng tiếp xúc $BC$. Áp dụng bổ đề 1 cho bộ 6 điểm $\{S, S, Z, X, T, Y\}$ suy ra $S, Z, X, T, Y$ đồng viên trên đường tròn tiếp xúc $BC$. Vậy kết thúc CM.

$$\begin{array}{| l | l |} \hline Ngockhanh99k48 & 8\\ \hline IHateMath & 1\\ \hline fatcat12345 & 5\\ \hline dogsteven & 5\\ \hline baopbc & 10\\ \hline QuangDuong12011998 & 5\\ \hline xuantrandong & 2\\ \hline mrjackass & 1\\ \hline vietnaminmyheart & 3\\ \hline BuiBaAnh & 1\\ \hline halloffame & 2\\ \hline Nguyen Dinh Hoang & 1\\ \hline TQHKTH & 1\\ \hline\end{array}$$

Hình gửi kèm

  • Bai dđth.png

Bài viết đã được chỉnh sửa nội dung bởi baopbc: 21-06-2016 - 09:51


#89
TQHKTH

TQHKTH

    Binh nhì

  • Thành viên mới
  • 11 Bài viết

Suy nghĩ lại, có lẽ mình nên CM lại cẩn thận bổ đề 2.

Chứng minh bổ đề 2. Gọi $S, U, V$ là tiếp điểm của inellipse với $BC, CA, AB$. $K, L$ là đối xứng của $P, Q$ qua $BA, BC$. Theo kết quả cơ bản, $\triangle BKQ = \triangle BPL$, suy ra $\angle BPV = \angle BKQ =\angle BPS$. Tương tự $\angle APU = \angle APV, \angle CPU = \angle CPS$. Do đó $\angle CPS+\angle APB = 180^\circ$.

Thế thì:

$\alpha = \angle PCB+\angle QBC+\angle PAB$

$= \angle PCB+(\angle PBA+\angle PAB)$

$= \angle PCB+ 180^\circ - \angle APB$

$= \angle PCB+\angle CPS$

$= \angle PSB$

Và bổ đề CM xong.

PS. Theo ý kiến mọi người, có lẽ mình sẽ đề nghị bài mới sau khi các bạn góp ý lời giải của mình :)

Hình gửi kèm

  • Lemma.png

Bài viết đã được chỉnh sửa nội dung bởi TQHKTH: 21-06-2016 - 09:25


#90
quanghung86

quanghung86

    Thiếu úy

  • Điều hành viên
  • 632 Bài viết

Trình bày lại latex cho Huy, lời giải trên đúng tuy nhiên phải dùng tới conic. Sau đây là lời giải dùng nghịch đảo

 

Giải. Chú ý rằng đường tròn $(M)$ qua $A,P$ và tiếp xúc $(PBC)$. Như vậy $(M)$ cắt $CA,AB$ tại $E',F'$ là đối xứng của $A$ qua $E,F$. Tương tự đường tròn $(N)$ qua $A,Q$ và tiếp xúc $(QBC)$ cắt $CA,AB$ tại $G',H'$ là đối xứng của $A$ qua $G,H$. Vậy ta cần chứng minh $E',F',G',H'$ cùng thuộc một đường tròn này tiếp xúc $BC$. Nghịch đảo cực $A$ phương tích bất kỳ. Chú ý rằng hai điểm đẳng giác vẫn có ảnh là hai điểm đẳng giác. Ta thu được bài toán sau của Bảo và Dương đã cm và phát biểu tại đây.

 

Cho tam giác $ABC$ có $P,Q$ là hai điểm đẳng giác. Tiếp tuyến tại $P$ của $(PBC)$ cắt $CA,AB$ tại $E,F$. Tiếp tuyến tại $Q$ của $(QBC)$ cắt $CA,AB$ tại $G,H$. Thì $E,F,G,H$ cùng thuộc một đường tròn tiếp xúc $(O)$.



#91
TQHKTH

TQHKTH

    Binh nhì

  • Thành viên mới
  • 11 Bài viết

Lời giải của thầy Hùng hay quá. Quả thực cấu hình ban đầu quá đẹp nên cũng ko nghĩ đến nghịch đảo, lại càng ko nghĩ nó liên quan đến bài cũ :)

Bây giờ mình xin tiếp tục Marathon với 1 bài toán nhẹ nhàng:

$\boxed{\text{Bài 42}}$ (Own) Cho $\triangle ABC$. Đường tròn nội tiếp $(I)$ tiếp xúc $BC, CA, AB$ tại $D, E, F$ tương ứng. $M$ là điểm bất kì trên $IA$. $X, Y$ là hình chiếu của $E, F$ lên $MC, MB$. Dựng hình bình hành $DXEK$ và $DYFH$. Đường tròn $(DKH)$ cắt lại $DI$ tại $T$. CMR: $MT\parallel AD$.

Hình gửi kèm

  • Marathon.png


#92
Ngockhanh99k48

Ngockhanh99k48

    Trung sĩ

  • Thành viên
  • 127 Bài viết

$\boxed{\text{Lời giải bài toán 42}}$
$DH$ và $DK$ thứ tự cắt $MB$ và $MC$ tại $P$ và $N$. $DF$ cắt $BM$ và $BI$ tại $R$ và $Z$. $DE$ cắt $CM$ và $CI$ tại $S$ và $Q$. $U$ là hình chiếu của $M$ trên $ID$. $AI$ cắt $BC$ tại $V$.
Ta có $A, M, I$ thẳng hàng nên $B(AMIC)=C(AMIB)$, suy ra $(FRZD)=(ESQD)$. Do đó $EF, RS, ZQ$ đồng quy hoặc đôi một song song. Ta lại có $Z, Q$ thứ tự là trung điểm $DF, DE$ nên $ZQ \parallel EF$. Do đó $RS \parallel EF$. Chú ý $DKEX$ và $DHFY$ là các hình bình hành nên $DP \perp MB$ và $DN \perp MC$. Mặt khác $\frac{\overline{DH}}{\overline{DP}}=\frac{-\overline{FY}}{\overline{DP}}=\frac{-\overline{RF}}{\overline{RD}}=\frac{-\overline{SE}}{\overline{SD}}=\frac{-\overline{EX}}{\overline{DN}}=\frac{\overline{DK}}{\overline{DN}}$. Do đó $HK \parallel NP$. Suy ra $\widehat{DTH}=\widehat{DKH}=\widehat{DNP}=\widehat{DUP}$. Do đó $TH \parallel UP$ và tương tự $TK \parallel UN$. Từ đó ta có $\frac{\overline{DT}}{\overline{DI}}=\frac{\overline{DT}}{\overline{DU}}.\frac{\overline{DU}}{\overline{DI}}=\frac{\overline{DH}}{\overline{DP}}.\frac{\overline{DU}}{\overline{DI}}=\frac{-\overline{RF}}{\overline{RD}}.\frac{\overline{VM}}{\overline{VI}}=\frac{\overline{RF}}{\overline{RD}}:\frac{\overline{ZF}}{\overline{ZD}}.\frac{\overline{VM}}{\overline{VI}}=(FDRZ).\frac{\overline{VM}}{\overline{VI}}=(AVMI).\frac{\overline{VM}}{\overline{VI}}=\frac{\overline{MA}}{\overline{MV}}.\frac{\overline{IV}}{\overline{IA}}.\frac{\overline{VM}}{\overline{VI}}=\frac{\overline{AM}}{\overline{AI}}$. Do đó $MT \parallel AD$. Ta có đpcm.

$$\begin{array}{| l | l |} \hline Ngockhanh99k48 & 9\\ \hline IHateMath & 1\\ \hline fatcat12345 & 5\\ \hline dogsteven & 5\\ \hline baopbc & 10\\ \hline QuangDuong12011998 & 5\\ \hline xuantrandong & 2\\ \hline mrjackass & 1\\ \hline vietnaminmyheart & 3\\ \hline BuiBaAnh & 1\\ \hline halloffame & 2\\ \hline Nguyen Dinh Hoang & 1\\ \hline TQHKTH & 1\\ \hline\end{array}$$


Bài viết đã được chỉnh sửa nội dung bởi baopbc: 22-06-2016 - 17:36
Up điểm


#93
Ngockhanh99k48

Ngockhanh99k48

    Trung sĩ

  • Thành viên
  • 127 Bài viết

$\boxed{\text{Bài toán 43(AoPS)}}$ Cho tam giác $\triangle ABC$ ngoại tiếp $(I)$. $D, E, F$ là tiếp điểm của $(I)$ với $BC, CA, AB$. $P$ là điểm bất kì. $DP, EP, FP$ thứ tự cắt $(I)$ tại điểm thứ hai $P_a, P_b, P_c$. 

a) Chứng minh $AP_a, BP_b, CP_c$ đồng quy tại $Q$.

b) Gọi $R, S$ là liên hợp đẳng giác của $P, Q$ đối với $\triangle ABC$. Chứng minh $RS$ đi qua điểm cố định. 


Bài viết đã được chỉnh sửa nội dung bởi Ngockhanh99k48: 23-06-2016 - 21:38


#94
Ngockhanh99k48

Ngockhanh99k48

    Trung sĩ

  • Thành viên
  • 127 Bài viết

$\boxed{\text{Lời giải bài toán 41}}$

Dễ thấy đường tròn $(M; MA)$ đi qua $A, M$ và tiếp xúc $(PBC)$, cắt $CA, AB$ thứ tự tại $E', F'$ là đối xứng của $A$ qua $E$ và $F$. Tương tự đường tròn $(N; NA)$ đi qua $A, Q$ và tiếp xúc $(QBC)$, cắt $CA, AB$ thứ tự tại $G', H'$ là đối xứng của $A$ qua $G$ và $H$. 

Vị tự tâm $A$ tỉ số 2, ta đưa bài toán về chứng minh $E', F', G', H'$ cùng thuộc một đường tròn tiếp xúc $BC$. 

Bổ đề: Cho hai đường tròn $(O_1), (O_2)$ tiếp xúc nhau tại $K$. $X, Y \in (O_1)$ và $Z, T \in (O_2)$ bất kì. Khi đó $(KY, KZ) \equiv (XY, XK) + (TK, TZ)$ (mod $\pi$).

Quay lại bài toán:

$(BPF')$ cắt $BC$ tại điểm thứ hai $X$. Áp dụng định lí Miquel cho $\triangle ABC$ với bộ 3 điểm ${X, E', F'}$ ta có $C, X, P, E'$ đồng viên. 

$(APB)$ cắt $(AQC)$ tại điểm thứ hai $Y$. Ta có $(YP, YC) \equiv (YP, YA) + (YA, YC) \equiv (BP, BA) + (QA, QC) \equiv (BP, BA) + (AQ, AC) + (CA, CQ) \equiv (BP, BA) + (AB, AP) + (CP, CB) \equiv (BP, BA) + (PF', PB)$ (áp dụng bổ đề cho $(BPC)$ và $(M; MA)$ tiếp xúc tại $P$) $\equiv (PF', BA) \equiv (F'P, F'A) \equiv (E'P, E'A) \equiv (E'P, E'C)$ (mod $\pi$). Do đó $C, X, P, Y, E'$ đồng viên. Ta lại có: $(YQ, YB) \equiv (YQ, YA) + (YA, YB) \equiv (CQ, CA) + (PA, PB) \equiv (CQ, CA) + (AP, AB) + (BA, BP) \equiv (CQ, CA) + (AC, AQ) + (BQ, BC) \equiv (CQ, CA) + (QG', QC)$ (áp dụng bổ đề cho $(BQC)$ và $(N; NA)$ tiếp xúc tại $Q$) $\equiv (QG', CA) \equiv (G'Q, G'A) \equiv (H'Q, H'A) \equiv (H'Q, H'B)$ (mod $\pi$). Do đó $B, Q, Y, H'$ đồng viên. Mặt khác $(YP, YQ) \equiv (YP, YA) + (YA, YQ) \equiv (BP, BA) + (CA, CQ) \equiv (BC, BQ) + (PC, BC) \equiv (PC, BQ)$ (mod $\pi$), suy ra $(QY, QB) \equiv (PY, PC) \equiv (XY, XC) \equiv (XY, XB)$ (mod $\pi$) hay $B, X, Q, Y, H'$ đồng viên. 

Chứng minh tương tự như trên ta có $(AQB), (APC), (BPF'), (CQG')$ đồng quy tại điểm $Z$. 

Ở trên ta đã có $(QA, QC) \equiv (PF', PB)$ (mod $\pi$) và $(PA, PB) \equiv (QG', QC)$ (mod $\pi$). Do đó $(E'F', E'G') \equiv (E'F', E'A) \equiv (PF', PA) \equiv (PF', PB) + (PB, PA) \equiv (QA, QC) + (QC, QG') \equiv (QA, QG') \equiv (H'A, H'G') \equiv (H'F', H'G')$ (mod $\pi$). Do đó $E'. F', G', H'$ đồng viên. Hơn nữa $(E'F', E'G') \equiv (PF', PB) + (PB, PA) \equiv (PF', PB) + (QC, QG') \equiv (XF', XB) + (XC, XG') \equiv (XF', XG')$ (mod $\pi$) nên $X \in (E'F'G'H')$. Mặt khác, $(XE', XC) \equiv (PE', PC) \equiv (AC, AP) + (BP, BC)$ (áp dụng bổ đề cho $(M; MA)$ và $(BPC)$ tiếp xúc) $\equiv (F'E', F'P) + (F'P, F'X) \equiv (F'E', F'X)$ (mod $\pi$), do đó $(E'F'G'H')$ tiếp xúc $BC$ tại điểm $X$. Ta hoàn tất chứng minh.



#95
baopbc

baopbc

    Himura Kenshin

  • Thành viên nổi bật 2016
  • 410 Bài viết

Ủng hộ câu a, trước. Câu b, chắc để sau vậy! :)

$\boxed{\text{Lời giải bài toán 43a}}$ Ta sẽ sử dụng bổ đề $Ceva$ trong đường tròn.

Cho tam giác $ABC$ nội tiếp $(O)$, các điểm $A',B',C'$ nằm trên $(O)$. Khi đó $AA',BB',CC'$ đồng quy $\Longleftrightarrow \frac{A'B}{A'C}.\frac{B'C}{B'A}.\frac{C'A}{C'B}=1$

Quay lại bài toán: Biến đổi tỉ số đơn giản $\frac{\sin \angle P_aAE}{\sin \angle P_aAF}=\frac{P_aE^2}{P_aF^2}$

Tương tự và áp dụng bổ đề trên ta suy ra $\frac{\sin \angle P_aAE}{\sin \angle P_aAF}.\frac{\sin \angle P_bBF}{\sin \angle P_bBD}.\frac{\sin \angle P_cCD}{\sin \angle P_cCE}=1$

Theo định lí $Ceva$ dạng lượng giác suy ra $AP_a,BP_b,CP_c$ đồng quy. $\blacksquare$

Post 222.png

PS. Anh Khánh ghi nguồn đầy đủ đi chứ? :mellow:


Bài viết đã được chỉnh sửa nội dung bởi baopbc: 23-06-2016 - 19:03


#96
QuangDuong12011998

QuangDuong12011998

    Hạ sĩ

  • Thành viên
  • 50 Bài viết

Bài 43 thì lời giải của mình dùng tỉ số kép.

 

Ý a là định lý Steinbart nổi tiếng.

Ý b khó ở chỗ là tìm điểm cố định, và điểm cố định là điểm đẳng giác của điểm Gergonne(điểm đồng quy của $AD$, $BE$, $CF$), tức là tâm vị tự trong $T$ của đường tròn nội tiếp và ngoại tiếp $\triangle ABC$.

Để ai đọc cũng có thể hiểu thì mình trình bày ngược.

$T$, $R$, $S$ thẳng hàng khi và chỉ khi $(BC,BT,BR,BS)=(CB,CT,CR,CS)$

$G$, $P$, $Q$ đẳng giác với $T$, $R$, $S$ nên

\[(BA,BG,BP,BQ)=(BC,BT,BR,BS)\text{ (đối xứng qua phân giác góc $B$)}\]

\[(CA,CG,CP,CQ)=(CB,CT,CR,CS)\text{ (đối xứng qua phân giác góc $C$)}\]

Như vậy $T$, $R$, $S$ thẳng hàng khi và chỉ khi $(BA,BG,BP,BQ)=(CA,CG,CP,CQ)$, chỉ cần chứng minh được đẳng thức tỉ số kép này.

Cho $PE$, $PF$ cắt $AB$, $AC$ tại $N$, $M$

\[(BA,BG,BP,BQ)=(BN,BE,BP,BP_b)=(N,E,P,P_b)\qquad (CA,CG,CP,CQ)=(CM,CF,CP,CP_c)=(M,F,P,P_c)\]

Biến đổi tương đương

\[(N,E,P,P_b)=(FN,FE,FP,FP_b)=(FF,FE,FP_c,FP_b)=(EF,EE,EP_c,EP_b)=(EE,EF,EP_b,EP_c)=(EM,EF,EP,EP_c)=(M,F,P,P_c)\]

Vậy chứng minh hoàn tất, $RS$ đi qua đẳng giác của điểm Gergonne.

P/S: Thầy và các bạn kiểm tra giúp em lời giải. Và thầy giúp em đề nghị bài mới được không ạ?

Bạn Khánh gửi link gốc được không?

 

Hình gửi kèm

  • X7 (1).png

Bài viết đã được chỉnh sửa nội dung bởi QuangDuong12011998: 24-06-2016 - 14:59


#97
quanghung86

quanghung86

    Thiếu úy

  • Điều hành viên
  • 632 Bài viết

Bài toán 43 gốc ở đây http://artofproblems...1198358p6114585 problem 13 của Telv Cohl.

 

Vừa rồi có 2 bài ELMO 2016 mới có trên mạng rất hay, nhờ cảm hứng bài này và được sự đồng ý của Dương, mình đề nghị bài sau

 

$\boxed{\text{Bài toán 44.}}$ Cho tam giác $ABC$ với $P$ là một điểm nằm trên trung trực $BC$ và $Q$ là đẳng giác của $P$ trong tam giác $ABC$. $PD$ là đường kính của đường tròn $(PBC)$. $QB,QC$ cắt đường tròn $(QCA),(QAB)$ tại $E,F$ khác $Q$. $K$ là tâm $(DEF)$. Chứng minh rằng $AK\perp BC$.

 



#98
baopbc

baopbc

    Himura Kenshin

  • Thành viên nổi bật 2016
  • 410 Bài viết

$\boxed{\text{Lời giải bài toán 44}}$ Bài gốc hay bài tổng quát đều không khó, lời giải của em dựa trên lời giải bài gốc mà em đưa lên trong link! :)

Do $P$ thuộc trung trực $BC$ nên $\angle QCA=\angle QBA\implies AE=AB,AF=AC\implies \triangle ABF=\triangle AEC$

$\implies BF=CE$. Mặt khác dễ thấy $\angle ECD=\angle DBF$ nên $\triangle DCE=\triangle DBF\implies DE=DF,\angle EDF=\angle BDC$

$\implies \angle EFD=\angle CBD.S\equiv DB\cap (DEF)$. Do $\angle BSE=\angle DFE=\angle DBC$ nên $BC\parallel SE$

Mặt khác dễ thấy $\angle BAE=2\angle DBC=2\angle BSE$ nên $A$ là tâm ngoại tiếp tam giác $BES\implies AK\perp SE\implies AK\perp BC.\blacksquare$



#99
quanghung86

quanghung86

    Thiếu úy

  • Điều hành viên
  • 632 Bài viết

Cám ơn Bảo, lời giải rất chuẩn, bài gốc thực chất không khó nhưng rất mới và đẹp, thầy rất thích những bài nhẹ nhàng và mới như thế :)! Từ bài tổng quát thầy vẫn thử tìm tổng quát hơn cho đẳng giác bất kỳ mà chưa thu được gì ? 

 

Nếu Bảo đồng ý thầy sẽ bù lại một bài đỡ hơn :)?



#100
quanghung86

quanghung86

    Thiếu úy

  • Điều hành viên
  • 632 Bài viết

Bảo đồng ý, mình đề nghị một bài sau

 

$\boxed{\text{Bài toán 45.}}$ Cho tam giác $ABC$ nội tiếp đường tròn $(O)$ và tâm nội tiếp $I$. Đường tròn $(K)$ tiếp xúc $CA,AB$ tại $E,F$ và tiếp xúc trong $(O)$ tại $D$. Gọi $M,N$ là tâm ngoại tiếp tam giác $ADE,ADF$. Trung trực $CA,AB$ cắt $BC$ tại $S,T$. Trên trung trực $IA$ lấy các điểm $P,Q$ sao cho $SP\parallel AC$ và $TQ\parallel AB$. Chứng minh rằng $MP,NQ$ và $AO$ đồng quy.

Post 229.png

Rất mong topic sẽ sôi nổi, tiến tới nếu đạt mốc 100 bài nên ra một ấn phẩm nhỏ kỷ niệm :)!


Bài viết đã được chỉnh sửa nội dung bởi baopbc: 25-06-2016 - 22:12
Up hộ thầy cái hình






Được gắn nhãn với một hoặc nhiều trong số những từ khóa sau: hình học

1 người đang xem chủ đề

0 thành viên, 1 khách, 0 thành viên ẩn danh